Saltar al contenido principal
LibreTexts Español

3.4: Interpretación de Integrales de Flujo

  • Page ID
    119157
  • \( \newcommand{\vecs}[1]{\overset { \scriptstyle \rightharpoonup} {\mathbf{#1}} } \) \( \newcommand{\vecd}[1]{\overset{-\!-\!\rightharpoonup}{\vphantom{a}\smash {#1}}} \)\(\newcommand{\id}{\mathrm{id}}\) \( \newcommand{\Span}{\mathrm{span}}\) \( \newcommand{\kernel}{\mathrm{null}\,}\) \( \newcommand{\range}{\mathrm{range}\,}\) \( \newcommand{\RealPart}{\mathrm{Re}}\) \( \newcommand{\ImaginaryPart}{\mathrm{Im}}\) \( \newcommand{\Argument}{\mathrm{Arg}}\) \( \newcommand{\norm}[1]{\| #1 \|}\) \( \newcommand{\inner}[2]{\langle #1, #2 \rangle}\) \( \newcommand{\Span}{\mathrm{span}}\) \(\newcommand{\id}{\mathrm{id}}\) \( \newcommand{\Span}{\mathrm{span}}\) \( \newcommand{\kernel}{\mathrm{null}\,}\) \( \newcommand{\range}{\mathrm{range}\,}\) \( \newcommand{\RealPart}{\mathrm{Re}}\) \( \newcommand{\ImaginaryPart}{\mathrm{Im}}\) \( \newcommand{\Argument}{\mathrm{Arg}}\) \( \newcommand{\norm}[1]{\| #1 \|}\) \( \newcommand{\inner}[2]{\langle #1, #2 \rangle}\) \( \newcommand{\Span}{\mathrm{span}}\)\(\newcommand{\AA}{\unicode[.8,0]{x212B}}\)

    Definimos, en §3.3, dos tipos de integrales sobre superficies. Hemos visto, en §3.3.4, algunas aplicaciones que conducen a integrales del tipo Ahora\(\iint_S \rho\,\text{d}S\text{.}\) miramos una aplicación que lleva a integrales del tipo\(\iint_S \vecs{F} \cdot\hat{\textbf{n}}\,\text{d}S\text{.}\) Recordemos que integrales de este tipo se denominan integrales de flujo. Imagina un fluido con

    • la densidad del fluido (digamos en kilogramos por metro cúbico) en la posición\((x,y,z)\) y el tiempo\(t\) siendo\(\rho(x,y,z,t)\) y con
    • la velocidad del fluido (digamos en metros por segundo) en la posición\((x,y,z)\) y el\(t\) tiempo\(\vecs{v} (x,y,z,t)\text{.}\)

    Vamos a determinar la velocidad (digamos en kilogramos por segundo) a la que el fluido fluye a través de un pequeño trozo\(\text{d}S\) de superficie en\((x,y,z)\text{.}\) Durante un pequeño intervalo de tiempo de duración\(\text{d}t\) aproximadamente el tiempo\(t\text{,}\) fluido cerca\(\text{d}S\) se mueve\(\vecs{v} (x,y,z,t)\text{d}t\text{.}\) La línea verde en la figura de abajo es un lado vista de\(\text{d}S\) y\(\hat{\textbf{n}}=\hat{\textbf{n}}(x,y,z)\) es un vector normal de unidad para\(\text{d}S\text{.}\)

    fluxIntDeriv.svg

    Así que durante ese pequeño intervalo de tiempo

    • la línea roja se mueve a la línea verde y
    • la línea verde se mueve a la línea azul para que
    • el fluido que llena la región gris oscuro debajo de la línea verde cruza\(\text{d}S\) y se mueve a la región gris claro por encima de la línea verde.

    Si denotamos por\(\theta\) el ángulo entre\(\hat{\textbf{n}}\) y\(\vecs{v} \text{d}t\text{,}\)

    • el volumen de fluido que atraviesa\(\text{d}S\) durante el intervalo de tiempo\(\text{d}t\) es el volumen cuya vista lateral es la región gris oscuro debajo de la línea verde. Esta región tiene base\(\text{d}S\) y altura\(|\vecs{v} \text{d}t|\cos\theta\) y así tiene volumen

      \[ |\vecs{v} (x,y,z,t)\text{d}t|\cos\theta\ \text{d}S =\vecs{v} (x,y,z,t)\cdot\hat{\textbf{n}}(x,y,z)\,\text{d}t\,\text{d}S \nonumber \]

      porque\(\hat{\textbf{n}}(x,y,z)\) tiene longitud uno.
    • La masa de fluido que cruza\(\text{d}S\) durante el intervalo de tiempo\(\text{d}t\) es entonces

      \[ \rho(x,y,z,t)\vecs{v} (x,y,z,t)\cdot\hat{\textbf{n}}(x,y,z)\,\text{d}t\,\text{d}S \nonumber \]

    • y la velocidad a la que el fluido está atravesando\(\text{d}S\) es

      \[ \rho(x,y,z,t)\vecs{v} (x,y,z,t)\cdot\hat{\textbf{n}}(x,y,z)\,\text{d}S \nonumber \]

    Integrando\(\text{d}S\) sobre una superficie\(S\text{,}\) concluimos que

    Lema 3.4.1

    La velocidad a la que la masa de fluido está cruzando a través de una superficie\(S\) es la integral del flujo

    \[ \iint_S \rho(x,y,z,t)\vecs{v} (x,y,z,t)\cdot\hat{\textbf{n}}(x,y,z)\,\text{d}S \nonumber \]

    Aquí\(\rho\) está la densidad del fluido,\(\vecs{v} \) es el campo de velocidad del fluido, y\(\hat{\textbf{n}}(x,y,z)\) es una unidad normal a\(S\) at\((x,y,z)\text{.}\) Si la integral de flujo es positiva el fluido está cruzando en la dirección\(\hat{\textbf{n}}\text{.}\) Si es negativo el fluido está cruzando opuesto a la dirección de \(\hat{\textbf{n}}\text{.}\)La velocidad a la que el volumen de fluido está cruzando a través de una superficie\(S\) es la integral del flujo

    \[ \iint_S \vecs{v} (x,y,z,t)\cdot\hat{\textbf{n}}(x,y,z)\,\text{d}S \nonumber \]

    Ejemplos de Integrales de Flujo

    Ejemplo 3.4.2. Fuente puntual

    En el Ejemplo 2.1.2, encontramos que el campo vectorial de una fuente puntual 1 (en tres dimensiones) que crea\(4\pi m\) litros por segundo es

    \[ \vecs{v} (x,y,z) = \frac{m}{r(x,y,z)^2}\,\hat{\textbf{r}} (x,y,z) \nonumber \]

    donde

    \[ r(x,y,z) = \sqrt{x^2+y^2+z^2}\qquad \hat{\textbf{r}} (x,y,z) = \frac{x\hat{\pmb{\imath}} + y\hat{\pmb{\jmath}} + z\hat{\mathbf{k}}}{\sqrt{x^2+y^2+z^2}} \nonumber \]

    Lo dibujamos en la Figura 2.1.3. Ahora calcularemos el flujo de este campo vectorial a través de una esfera centrada en el origen. Supongamos que la esfera tiene radio\(R\text{.}\)

    sourceFlux.svg

    Entonces el hacia afuera 2 apuntando normal en un punto\((x,y,z)\) de la esfera es

    \[ \hat{\textbf{n}}(x,y,z) = \hat{\textbf{r}} (x,y,z) = \frac{x\hat{\pmb{\imath}} + y\hat{\pmb{\jmath}} + z\hat{\mathbf{k}}}{\sqrt{x^2+y^2+z^2}} = \frac{x\hat{\pmb{\imath}} + y\hat{\pmb{\jmath}} + z\hat{\mathbf{k}}}{R} \nonumber \]

    Tenga en cuenta que\(\hat{\textbf{r}} (x,y,z)\cdot \hat{\textbf{r}} (x,y,z)=1\) y que, en la esfera,\(r(x,y,z)=R\text{.}\) Así que el flujo del\(\vecs{v} \) exterior a través de la esfera es

    \[\begin{align*} \iint_S\vecs{v} \cdot\hat{\textbf{n}}\ \text{d}S &= \iint_S \frac{m}{r(x,y,z)^2}\,\hat{\textbf{r}} (x,y,z) \cdot \hat{\textbf{r}} (x,y,z)\ \text{d}S\\ &= \iint_S \frac{m}{R^2}\ \text{d}S =\frac{m}{R^2} 4\pi R^2\\ &=4\pi m \end{align*}\]

    Esta es la velocidad a la que el volumen de fluido sale de la esfera. En nuestra derivación del campo vectorial asumimos que el fluido es incompresible, por lo que también es la velocidad a la que la fuente puntual está creando fluido.

    Ejemplo 3.4.3. Vortex

    En la Figura 2.1.6, esbozamos el campo vectorial (en dos dimensiones)

    \[ \vecs{v} (x,y) = \Omega\big(-y\hat{\pmb{\imath}} +x\hat{\pmb{\jmath}}\big) \nonumber \]

    Ahora calcularemos el flujo de este campo vectorial a través de un círculo\(C\) centrado en el origen. Supongamos que el círculo tiene radio\(R\text{.}\)

    vortexFlux.svg

    Por definición, en dos dimensiones, el flujo de un campo vectorial a través de una curva\(C\) es\(\int_C\vecs{v} \cdot\hat{\textbf{n}}\ \text{d}s\text{.}\)

    Este es el análogo natural del flujo en tres dimensiones: la superficie\(S\) ha sido reemplazada por la curva\(C\text{,}\) y la superficie\(\text{d}S\) de una pequeña pieza de\(S\) ha sido reemplazada por la longitud\(ds\) del arco de una pequeña pieza de\(C\text{.}\)

    La unidad que apunta hacia afuera normal en un punto\((x,y)\) de nuestro círculo\(C\) es

    \[ \hat{\textbf{n}}(x,y) = \frac{x\hat{\pmb{\imath}} + y\hat{\pmb{\jmath}}}{\sqrt{x^2+y^2}} = \frac{x\hat{\pmb{\imath}} + y\hat{\pmb{\jmath}}}{R} \nonumber \]

    Entonces

    \[ \vecs{v} (x,y)\cdot\hat{\textbf{n}}(x,y) =\frac{\Omega}{R}\big(-y\hat{\pmb{\imath}} + x\hat{\pmb{\jmath}}\big)\cdot\big(x\hat{\pmb{\imath}} + y\hat{\pmb{\jmath}}\big) =0 \nonumber \]

    y el flujo a través\(C\) es

    \[ \int_C\vecs{v} \cdot\hat{\textbf{n}}\ \text{d}s=0 \nonumber \]

    Esto no debería ser una sorpresa — ningún fluido está cruzando\(C\) en absoluto. Esto es exactamente lo que esperaríamos al mirar las flechas en la Figura 2.1.6 o en las líneas del arroyo en el Ejemplo 2.2.6.

    Ejemplo 3.4.4

    Evaluar\(\ \iint_S\vecs{F} \cdot\hat{\textbf{n}}\ dS\ \) dónde

    \[ \vecs{F} (x,y,z) = (x+y)\,\hat{\pmb{\imath}} + (y+z)\,\hat{\pmb{\jmath}} + (x+z)\,\hat{\mathbf{k}} \nonumber \]

    y\(S\) es el límite de\(V=\left \{(x,y,z)|0\le x^2+y^2\le 9,\ 0\le z\le 5 \right \}\text{,}\) y\(\hat{\textbf{n}}\) es el exterior normal 3 a\(S\text{.}\)

    Solución

    El volumen\(V\) parece una lata de radio\(3\) y altura\(5\text{.}\)

    cylinder1.svg

    Es natural descomponer su superficie\(S\) en tres partes

    \[\begin{align*} S_t &= \left \{(x,y,z)|0\le x^2+y^2\le 9,\ z= 5 \right \} = \text{the top}\\ S_b &= \left \{(x,y,z)|0\le x^2+y^2\le 9,\ z= 0 \right \} = \text{the bottom}\\ S_s &= \left \{(x,y,z)|x^2+y^2 = 9,\ 0\le z\le 5 \right \} = \text{the side} \end{align*}\]

    Calcularemos el flujo a través de cada una de las tres partes por separado y luego las agregaremos juntas.

    The Top: En la parte superior, lo que apunta hacia afuera normal a\(S\) es\(\hat{\textbf{n}}=\hat{\mathbf{k}}\) y\(\text{d}S = \text{d}x\text{d}y\text{.}\) Esto es probablemente intuitivamente obvio. Pero si no lo es, siempre puedes derivarlo parametrizando la parte superior por\(\vecs{r} (x,y) = x\,\hat{\pmb{\imath}} +y\,\hat{\pmb{\jmath}} + 5\,\hat{\mathbf{k}}\) con\(x^2+y^2\le 9\text{.}\) Entonces el flujo a través de la parte superior es

    \[\begin{align*} \iint_{S_t}\vecs{F} \cdot\hat{\textbf{n}}\ \text{d}S &= \iint_{x^2+y^2\le 9\\z=5} (x+z)\ \text{d}x\text{d}y = \iint_{x^2+y^2\le 9} (x+5)\ \text{d}x\text{d}y \end{align*}\]

    La integral\(\iint_{x^2+y^2\le 9} x\ \text{d}x\text{d}y=0\) ya\(x\) es impar y el dominio de la integración es simétrico sobre\(x=0\text{.}\) So

    \[\begin{gather*} \iint_{S_t}\vecs{F} \cdot\hat{\textbf{n}}\ \text{d}S = \iint_{x^2+y^2\le 9} 5\ \text{d}x\text{d}y =5\pi(3)^2 =45\pi \end{gather*}\]

    La parte inferior: En la parte inferior, el hacia afuera apuntando normal a\(S\) es\(\hat{\textbf{n}}=-\hat{\mathbf{k}}\) y\(\text{d}S = \text{d}x\text{d}y\text{.}\) así el flujo a través de la parte inferior es

    \[\begin{align*} \iint_{S_b}\vecs{F} \cdot\hat{\textbf{n}}\ \text{d}S &= -\iint_{x^2+y^2\le 9\\z=0} (x+z)\ \text{d}x\text{d}y = -\iint_{x^2+y^2\le 9} x\ \text{d}x\text{d}y =0 \end{align*}\]

    de nuevo ya que\(x\) es impar y el dominio de la integración es simétrico sobre\(x=0\text{.}\)

    El Lado: Podemos parametrizar el lado mediante el uso de coordenadas cilíndricas.

    \[ \vecs{r} (\theta,z) = \big(3\cos\theta\,,\,3\sin\theta\,,\,z\big)\qquad 0\le\theta \lt 2\pi,\ 0\le z\le 5 \nonumber \]

    Luego, usando 3.3.1,

    canTopView.svg

    \[\begin{align*} \frac{\partial\vecs{r} }{\partial\theta} &=(-3\sin\theta\,,\,3\cos\theta\,,\,0)\\ \frac{\partial\vecs{r} }{\partial z} &=(0\,,\,0\,,\,1)\\ \hat{\textbf{n}}\,\text{d}S&= \frac{\partial\vecs{r} }{\partial\theta} \times\frac{\partial\vecs{r} }{\partial z}\ \text{d}\theta\,\text{d}z\\ &=(3\cos\theta\,,\,3\sin\theta\,,\,0)\ \text{d}\theta\,\text{d}z \end{align*}\]

    Tenga en cuenta que\(\hat{\textbf{n}} = (\cos\theta\,,\,\sin\theta\,,\,0)\) está apuntando hacia afuera 4, según se desee. Continuando,

    \[\begin{align*} \vecs{F} \big(x(\theta,z),y(\theta,z),z(\theta,z)\big) &=3(\cos\theta\!+\!\sin\theta)\,\hat{\pmb{\imath}} +(3\sin\theta\!+\!z)\,\hat{\pmb{\jmath}}+(3\cos\theta\!+\!z)\,\hat{\mathbf{k}}\\ \vecs{F} \cdot\hat{\textbf{n}}\,\text{d}S &=\big\{9\cos^2\theta\!+\!3\sin\theta\cos\theta\!+\!9\sin^2\theta\!+\!3z\sin\theta\big\}\,\text{d}\theta\,\text{d}z\\ &=\big\{9 +\frac{3}{2}\,\sin(2\theta)+3z\sin\theta\big\}\ \text{d}\theta\,\text{d}z \end{align*}\]

    Entonces el flujo a través del lado es

    \[\begin{align*} \iint_{S_s}\vecs{F} \cdot\hat{\textbf{n}}\ \text{d}S &=\int_0^{2\pi}\text{d}\theta\int_0^5\text{d}z\ \big\{9 +\frac{3}{2}\,\sin(2\theta)+3z\sin\theta\big\}\\ &=9\int_0^{2\pi}\text{d}\theta\int_0^5\text{d}z \quad\text{since }\int_0^{2\pi}\sin\theta\,\text{d}\theta =\int_0^{2\pi}\sin(2\theta)\,\text{d}\theta =0\\ &=9\times 2\pi\times 5 =90\pi \end{align*}\]

    y el flujo total es

    \[\begin{align*} \iint_{S}\vecs{F} \cdot\hat{\textbf{n}}\ \text{d}S &=\iint_{S_t}\vecs{F} \cdot\hat{\textbf{n}}\ \text{d}S +\iint_{S_b}\vecs{F} \cdot\hat{\textbf{n}}\ \text{d}S +\iint_{S_s}\vecs{F} \cdot\hat{\textbf{n}}\ \text{d}S\\ &=45\pi+0+90\pi =135\pi \end{align*}\]

    Ejemplo 3.4.5

    Evaluar\(\ \iint_S\vecs{F} \cdot\hat{\textbf{n}}\ dS\ \) dónde\(\ \vecs{F} (x,y,z)=x^4\hat{\pmb{\imath}}+2y^2\hat{\pmb{\jmath}}+z\hat{\mathbf{k}},\ \)\(S\) está la mitad de la superficie\(\ \frac{1}{4}x^2+\frac{1}{9}y^2+z^2=1\ \) con\(z\ge 0\text{,}\) y\(\hat{\textbf{n}}\) es la unidad que apunta hacia arriba normal.

    ellipsoid.svg

    Solución 1

    Comenzamos por parametrizar la superficie, que es la mitad de un elipsoide. A modo de motivación para la parametrización, recordemos que las coordenadas esféricas, con\(\rho=1\text{,}\) proporcionan una forma natural de parametrizar la esfera\(x^2+y^2+z^2=1\text{.}\) A saber\(x=\cos\theta\sin\varphi\text{,}\)\(y=\sin\theta\sin\varphi\text{,}\)\(z= \cos\varphi\text{.}\) La razón por la que funcionan estas coordenadas esféricas es que la identidad trig\(\cos^2\alpha+\sin^2\alpha=1\) implica

    \[ x^2+y^2 = \cos^2\theta\sin^2\varphi + \sin^2\theta\sin^2\varphi =\sin^2\varphi \nonumber \]

    y luego

    \[ \big(x^2+y^2\big) + z^2 = \sin^2\varphi +\cos^2\varphi = 1 \nonumber \]

    La ecuación de nuestro elipsoide es

    \[ \Big(\frac{x}{2}\Big)^2 + \Big(\frac{y}{3}\Big)^2 + z^2 =1 \nonumber \]

    así podemos parametrizar el elipsoide reemplazando\(x\) con\(\frac{x}{2}\) y\(y\) con\(\frac{y}{3}\) en nuestra parametrización de la esfera. Es decir, elegimos la parametrización

    \[\begin{align*} x(\theta,\varphi)&=2\cos\theta\sin\varphi\\ y(\theta,\varphi)&=3\sin\theta\sin\varphi\\ z(\theta,\varphi)&=\cos\varphi \end{align*}\]

    con\((\theta,\varphi)\) atropello\(0\le\theta\le 2\pi,\ 0\le\varphi\le\pi/2\text{.}\) Tenga en cuenta que

    \[ \frac{1}{4}x(\theta,\varphi)^2+\frac{1}{9}y(\theta,\varphi)^2 +z(\theta,\varphi)^2=1 \nonumber \]

    según se desee.

    Luego, usando 3.3.1,

    \[\begin{align*} \Big(\frac{\partial x}{\partial\theta}\,,\,\frac{\partial y}{\partial\theta} \,,\, \frac{\partial z}{\partial\theta}\Big) &=(-2\sin\theta\sin\varphi\,,\,3\cos\theta\sin\varphi\,,\,0)\\ \Big(\frac{\partial x}{\partial\varphi}\,,\,\frac{\partial y}{\partial\varphi}\,,\, \frac{\partial z}{\partial\varphi}\Big) &=(2\cos\theta\cos\varphi\,,\,3\sin\theta\cos\varphi\,,\,-\sin\varphi)\\ \hat{\textbf{n}}\,\text{d}S&= -\Big(\frac{\partial x}{\partial\theta}\,,\,\frac{\partial y}{\partial\theta} \,,\, \frac{\partial z}{\partial\theta}\Big) \times\Big(\frac{\partial x}{\partial\varphi}\,,\,\frac{\partial y}{\partial\varphi} \,,\,\frac{\partial z}{\partial\varphi}\Big)\ \text{d}\theta \text{d}\varphi\\ &=-(-3\cos\theta\sin^2\varphi,-2\sin\theta\sin^2\varphi,-6\sin\varphi\cos\varphi)\text{d}\theta \text{d}\varphi \end{align*}\]

    El signo extra de menos\(\hat{\textbf{n}}\,\text{d}S\) se puso ahí para hacer que el\(z\) componente sea\(\hat{\textbf{n}}\) positivo. (El problema especificó que\(\hat{\textbf{n}}\) es ser ascendente unidad normal.) Como

    \[\begin{align*} &\vecs{F} \big(x(\theta,\varphi)\,,\,y(\theta,\varphi)\,,\,z(\theta,\varphi)\big)\\ &\hskip0.5in=2^4\cos^4\theta\sin^4\varphi\ \hat{\pmb{\imath}} +2\times 3^2\sin^2\theta\sin^2\varphi\ \hat{\pmb{\jmath}}+\cos\varphi\ \hat{\mathbf{k}} \end{align*}\]

    tenemos

    \[\begin{align*} \vecs{F} \cdot\hat{\textbf{n}}\,\text{d}S &=\Big[3\times 2^4\cos^5\theta\sin^6\varphi\!+\!2\times 2 \times 3^2 \sin^3\theta\sin^4\varphi\!+\!6\sin\varphi\cos^2\varphi\Big]\,\text{d}\theta\text{d}\varphi \end{align*}\]

    y la integral deseada

    \[\begin{align*} \iint_S\vecs{F} \cdot\hat{\textbf{n}}\ \text{d}S &=\int_0^{\frac{\pi}{2}}\!\!\text{d}\varphi\int_0^{2\pi}\text{d}\theta\ \Big[3\times 2^4\cos^5\theta\sin^6\varphi+2\times 2\times 3^2 \sin^3\theta\sin^4\varphi\\ &\hskip3in+6\sin\varphi\cos^2\varphi\Big] \end{align*}\]

    Dado que\(\ \int_0^{2\pi} \cos^m\theta\,\text{d}\theta =\int_0^{2\pi} \sin^m\theta\,\text{d}\theta=0\ \) para todos los números naturales impares 5\(m\text{,}\)

    \[\begin{align*} \iint_S \vecs{F} \cdot\hat{\textbf{n}}\, \text{d}S &=\int_0^{\pi/2}\hskip-8pt \text{d}\varphi\int_0^{2\pi}\hskip-6pt\text{d}\theta\ 6\sin\varphi\cos^2\varphi =12\pi\int_0^{\pi/2}\hskip-8pt \text{d}\varphi\ \sin\varphi\cos^2\varphi\\ &=12\pi\Big[-\frac{1}{3}\cos^3\varphi\Big]_0^{\pi/2} =4\pi \end{align*}\]

    La integral se evaluó adivinando (y comprobando) que\(-\frac{1}{3}\cos^3\varphi\) es un antiderivado de\(\sin\varphi\cos^2\varphi\text{.}\) También se puede hacer sustituyendo\(u=\cos\varphi\text{,}\)\(\text{d}u=-\sin\varphi\,\text{d}\varphi\text{.}\)

    Solución 2

    Esta vez parametrizaremos el semielipsoide usando una variante de coordenadas cilíndricas.

    \[\begin{align*} x(r,\theta)&=2r\cos\theta\\ y(r,\theta)&=3r\sin\theta\\ z(r,\theta)&=\sqrt{1-r^2} \end{align*}\]

    con\((r,\theta)\) atropello\(0\le\theta\le 2\pi,\ 0\le r\le1\text{.}\) Porque construimos los factores de\(2\)\(x(r,\theta)\) y\(3\) en y\(y(r,\theta)\text{,}\) tenemos

    \[\begin{align*} &\frac{x(r,\theta)^2}{4} + \frac{y(r,\theta)^2}{9} =r^2\cos^2\theta+r^2\sin^2\theta = r^2\\ \implies &\frac{x(r,\theta)^2}{4} + \frac{y(r,\theta)^2}{9} +z(r,\theta)^2 = r^2 + \left(\sqrt{1-r^2}\right)^2=1 \end{align*}\]

    según se desee. Además\(z(r,\theta)\ge 0\) por nuestra elección de raíz cuadrada en la definición de\(z(r,\theta)\text{.}\)

    Entonces, usando 3.3.1,

    \[\begin{align*} \Big(\frac{\partial x}{\partial \theta},\frac{\partial y}{\partial \theta}, \frac{\partial z}{\partial \theta}\Big) &=(-2r\sin\theta,3r\cos\theta,0)\\ \Big(\frac{\partial x}{\partial r},\frac{\partial y}{\partial r}, \frac{\partial z}{\partial r}\Big) &=\Big(2\cos\theta,3\sin\theta,-\frac{r}{\sqrt{1-r^2}}\Big)\\ \hat{\textbf{n}} \text{d}S&= -\Big(\frac{\partial x}{\partial\theta},\frac{\partial y}{\partial\theta}, \frac{\partial z}{\partial\theta}\Big) \times\Big(\frac{\partial x}{\partial r},\frac{\partial y}{\partial r}, \frac{\partial z}{\partial r}\Big)dr\,\text{d}\theta\\ &=-\Big(-\frac{3r^2\cos\theta}{\sqrt{1-r^2}}, -\frac{2r^2\sin\theta}{\sqrt{1-r^2}},-6r\Big)\text{d}r\,\text{d}\theta \end{align*}\]

    Una vez más,\(\hat{\textbf{n}} \text{d}S\) se puso ahí el signo extra de menos para hacer que el\(z\) componente sea\(\hat{\textbf{n}}\) positivo. Continuando,

    \[\begin{align*} \vecs{F} \big(x(r,\theta)\,,\,y(r,\theta)\,,\,z(r,\theta)\big) &=2^4r^4\cos^4\theta\,\hat{\pmb{\imath}}+2\times 3^2r^2\sin^2\theta\,\hat{\pmb{\jmath}}+\sqrt{1-r^2}\,\hat{\mathbf{k}}\\ \vecs{F} \cdot\hat{\textbf{n}}\, \text{d}S&=\Big[3\times2^4\frac{r^6}{\sqrt{1-r^2}}\cos^5\theta +2^2 3^2\frac{r^4}{\sqrt{1-r^2}}\sin^3\theta\\ &\hskip2.3in+6r\sqrt{1-r^2}\Big]\,\text{d}r\,\text{d}\theta \end{align*}\]

    Otra vez usando eso\(\ \int_0^{2\pi} \cos^m\theta\,\text{d}\theta =\int_0^{2\pi} \sin^m\theta\,\text{d}\theta=0\ \) para todos los números naturales impares\(m\text{,}\)

    \[\begin{align*} \int_S \vecs{F} \cdot\hat{\textbf{n}}\, \text{d}S &=\int_0^1 dr\int_0^{2\pi}\hskip-6pt\text{d}\theta\ 6r\sqrt{1-r^2}\\ &=12\pi\int_0^1 dr\ r\sqrt{1-r^2} =12\pi\Big[-\frac{1}{3}{(1-r^2)}^{3/2}\Big]_0^1\\ &=4\pi \end{align*}\]

    La integral se evaluó adivinando (y comprobando) que\(-\frac{1}{3}{(1-r^2)}^{3/2}\) es un antiderivado de\(r\sqrt{1-r^2}\text{.}\) También se puede hacer sustituyendo\(u=1-r^2\text{,}\)\(\text{d}u=-2r\,\text{d}r\text{.}\)

    Solución 3

    La superficie es de la forma\(G(x,y,z)=0\) con\(G(x,y,z)=\frac{1}{4}x^2+\frac{1}{9}y^2+z^2-1\text{.}\) Por lo tanto, usando 3.3.3,

    \[\begin{align*} \hat{\textbf{n}} \text{d}S&=\frac{\vecs{ \nabla} G}{\vecs{ \nabla} G\cdot\hat{\mathbf{k}}}\text{d}x\,\text{d}y =\frac{\frac{x}{2}\hat{\pmb{\imath}}+\frac{2y}{9}\hat{\pmb{\jmath}}+2z\hat{\mathbf{k}}}{2z}\text{d}x\,\text{d}y\\ &=\Big(\frac{x}{4z}\hat{\pmb{\imath}}+\frac{y}{9z}\hat{\pmb{\jmath}}+\hat{\mathbf{k}}\Big)\text{d}x\,\text{d}y\\ \implies \vecs{F} \cdot\hat{\textbf{n}}\,\text{d}S &=\Big(\frac{x^5}{4z}+\frac{2y^3}{9z}+z\Big)\text{d}x\,\text{d}y \end{align*}\]

    Es cierto que\(\hat{\textbf{n}}\text{d}S\text{,}\) y consecuentemente\(\vecs{F} \cdot\hat{\textbf{n}}\,\text{d}S\) llegar a ser infinito 6

    como\(z\rightarrow 0\text{.}\) Así que realmente deberíamos tratar a la integral como una integral impropia, integrándonos primero por encima\(z\ge\varepsilon\) y luego tomando el límite\(\varepsilon\rightarrow 0^+\text{.}\) Pero, como veremos, la singularidad es inofensiva. Por lo que es estándar pasar por alto este punto. En\(S\text{,}\)\(z=z(x,y)=\sqrt{1-\frac{x^2}{4}-\frac{y^2}{9}}\) y\(\frac{x^2}{4}+\frac{y^2}{9}\le 1\text{,}\) así

    \[\begin{align*} \int_S \vecs{F} \cdot\hat{\textbf{n}}\, \text{d}S &=\iint_{\frac{x^2}{4}+\frac{y^2}{9}\le 1} \Big(\frac{x^5}{4z(x,y)}+\frac{2y^3}{9z(x,y)}+z(x,y)\Big)\ \text{d}x\,\text{d}y \end{align*}\]

    Ambos\(\frac{x^5}{4z(x,y)}\) y\(\frac{2y^3}{9z(x,y)}\) son impares bajo\(x\rightarrow-x,\ y\rightarrow -y\) y el dominio de integración es par bajo\(x\rightarrow-x,\ y\rightarrow -y\text{,}\) por lo que sus integrales son cero y

    \[\begin{align*} \int_S \vecs{F} \cdot\hat{\textbf{n}}\, \text{d}S &=\iint_{\frac{x^2}{4}+\frac{y^2}{9}\le 1}z(x,y)\ \text{d}x\,\text{d}y\\ &=\iint_{\frac{x^2}{4}+\frac{y^2}{9}\le 1} \sqrt{1-\frac{x^2}{4}-\frac{y^2}{9}}\ \text{d}x\,\text{d}y \end{align*}\]

    Para evaluar esta integral, primero hacer el cambio de variables 7\(x=2X\text{,}\)\(\text{d}x=2\text{d}X\text{,}\)\(y=3Y\text{,}\)\(\text{d}y=3\text{d}Y\) para dar

    \[ \int_S \vecs{F} \cdot\hat{\textbf{n}}\, \text{d}S =\iint_{X^2+Y^2\le 1} \sqrt{1-X^2-Y^2}\ 6\,\text{d}X\,\text{d}Y \nonumber \]

    Luego cambia a coordenadas polares,\(X=r\cos\theta\text{,}\)\(Y=r\sin\theta\text{,}\)\(\text{d}X\text{d}Y = r\,\text{d}r\text{d}\theta\) para dar

    \[\begin{align*} \int_S \vecs{F} \cdot\hat{\textbf{n}}\, \text{d}S &=\int_0^1 dr\int_0^{2\pi}\hskip-6pt\text{d}\theta\ 6r\sqrt{1-r^2} =12\pi\int_0^1 dr\ r\sqrt{1-r^2}\\ &=12\pi\Big[-\frac{1}{3}{(1-r^2)}^{3/2}\Big]_0^1 =4\pi \end{align*}\]

    Solución 4

    La superficie es de la forma\(z=f(x,y)\) con\(f(x,y)=\sqrt{1-\frac{x^2}{4}-\frac{y^2}{9}}\text{.}\) Por lo tanto, usando 3.3.2,

    \[\begin{align*} \hat{\textbf{n}} \text{d}S &=\Big[-\frac{\partial f}{\partial x}\hat{\pmb{\imath}}\!-\!\frac{\partial f}{\partial y}\hat{\pmb{\jmath}} \!+\!\hat{\mathbf{k}}\Big]\,\text{d}x\,\text{d}y =\left[\frac{\frac{x}{4}\hat{\pmb{\imath}}+\frac{y}{9}\hat{\pmb{\jmath}}} {\sqrt{1-\frac{x^2}{4}-\frac{y^2}{9}}}+\hat{\mathbf{k}}\right]\text{d}x\,\text{d}y\cr \implies\! \vecs{F} \cdot\hat{\textbf{n}}\, \text{d}S&=\left[\frac{\frac{x^5}{4}+\frac{2y^3}{9}} {\sqrt{1-\frac{x^2}{4}-\frac{y^2}{9}}} +\sqrt{1-\frac{x^2}{4}-\frac{y^2}{9}}\right]\text{d}x\,\text{d}y \end{align*}\]

    Tenga en cuenta que nuestra unidad normal está apuntando hacia arriba, según sea necesario. Al igual que en la Solución 3, por la rareza de los\(y^3\) términos\(x^5\) y en el integrando,

    \[\begin{align*} \int_S \vecs{F} \cdot\hat{\textbf{n}}\, \text{d}S &=\iint_{\frac{x^2}{4}+\frac{y^2}{9}\le 1} \left[\frac{\frac{x^5}{4}+\frac{2y^3}{9}}{\sqrt{\ \cdots\ }} +\sqrt{1-\frac{x^2}{4}-\frac{y^2}{9}}\right]\text{d}x\,\text{d}y\\ &=\iint_{\frac{x^2}{4}+\frac{y^2}{9}\le 1} \sqrt{1-\frac{x^2}{4}-\frac{y^2}{9}}\ \text{d}x\,\text{d}y \end{align*}\]

    Ahora continúe como en la Solución 3.

    1. Se puede imaginar que una tubería muy pequeña bombea agua al origen.
    2. Realmente no importa qué unidad normal escojamos aquí. Sólo tenemos que tener claro cuál estamos usando. Con la normal hacia afuera, el flujo da la velocidad a la que el fluido cruza la esfera en la dirección hacia afuera. Si tuviéramos que usar la normal que apunta hacia adentro, el flujo daría la velocidad a la que el fluido cruza la esfera en la dirección hacia adentro.
    3. Es necesario que el problema especifique, de una forma u otra, si\(\hat{\textbf{n}}\) es la normal que apunta hacia adentro o la normal que apunta hacia afuera. Sin esto, el significado de\(\ \iint_S\vecs{F} \cdot\hat{\textbf{n}}\ dS\ \) es ambiguo. Piense en dónde se usa la orientación del vector normal en su solución.
    4. Para verificar, dibuja, en tu cabeza, un boceto de la vista superior de la lata. “Vista superior” solo significa “ignorar la\(z\) coordenada”. La vista superior de la lata es un círculo de radio\(3\text{.}\) Entonces, en un punto genérico,\(\vecs{r} =(\cos\theta,\sin\theta)\text{,}\) en la lata, dibuja la unidad normal\(\hat{\textbf{n}} = (\cos\theta\,,\,\sin\theta)\) con su cola en\(\vecs{r} \text{.}\) Está apuntando lejos del origen, al igual que lo\(\vecs{r} \) es. Es decir,\(\hat{\textbf{n}}\) está apuntando hacia afuera.
    5. Mira las gráficas de\(\cos^m\varphi\) y\(\sin^m\varphi\text{.}\)
    6. Eso es porque el elipsoide se está volviendo vertical como\(z\rightarrow 0\text{,}\) así así\(x\) y no\(y\) son realmente buenos parámetros ahí.
    7. El lector interesado en los cambios generales de variables en integrales multidimensionales debe buscar “determinante jacobiano”.

    This page titled 3.4: Interpretación de Integrales de Flujo is shared under a CC BY-NC-SA 4.0 license and was authored, remixed, and/or curated by Joel Feldman, Andrew Rechnitzer and Elyse Yeager via source content that was edited to the style and standards of the LibreTexts platform; a detailed edit history is available upon request.